Monday, August 23, 2010

What should i expect from the dermatologist?

i am very nervous about going for the first time, i hav acne scars all over my face which is officially ruined and i am terrified of the thought of going to someone to look at my nasty face without my makeup...i haven't gone without my makeup in public for three years!! Any advice?? Could anyone give me a thorough run through of how it will be??What should i expect from the dermatologist?
You will walk in and fill out your fourms in the waiting area. Then you will get called back and a medical assitant will do their usual questionare ';why are you here'; stuff then the doctor will enter. He/She will notice the issue immediately and ask you about it. They will put on gloves and examine the face closely.





After that is done, you can discuss treatment options for your scars.





Don't be worried about going to the derm. They are specially trained to help you with your skin issues and I am sure they have seen everything. I have been to 3 in my 4 year quest to fight acne and I am glad I went to a derm.





I hope this helped..best of wishes

What should I major in college if I want to be a Dermatologist (Doctor)?

What should I major in college if I want to be a Dermatologist (Doctor)?





And if i change my mind after i get my bachelors degree can i be a physician assistant?





P.S= how much money does a dermatologist make?What should I major in college if I want to be a Dermatologist (Doctor)?
Electric EngineeringWhat should I major in college if I want to be a Dermatologist (Doctor)?
The obvious answer would be to do your undergrad in Pre-Med. However, that may not be the best idea.





I was looking to enroll in Med school about 3 years ago when I was 30. I did my undergrad in Electrical Engineering and I was concerned about my undergrad choice having a negative impact on my chances of getting into Med School.





I called two Medical schools and talked with their admissions departments. Once of the things that I found out that really shocked me is that both of the schools I spoke with make it a point to go out of their way and recruit some students who were not Pre-Med majors. They do this to ensure a class of students with a diverse educational background, thus having a student body where people think differently. The schools still seeked students with undergrad majors from science fields, so having an EE degree met their requirements.





I was told that one of the most important factors in determining admission was a prospective students performance on the MCATs.





Personally, I'd pick about 5 medical schools you think you'd like to attend and call them. Find out what the admissions board looks for in their incoming student body. Based on what you find out, decide on a major that you enjoy and will find you good paying work in the event you don't make it in Med School or you change your mind. If you study Pre-Med and decide Med School isn't for you, what job prospects do you have? Maybe a 4 year BS degree in Nursing might get you into Med School yet at the same time offer a great safety net if you change your mind.





In terms of getting a PA degree, a PA degree is a Master's program where a MD/DO is a Doctorate degree. You can get a PA degree in about 24 to 30 months if you go full time (whereas a MD/DO is 4 years of schooling, up to several years for your specialty training, plus several years as a resident where you get worked to death for $40K a year).





The money for being a doctor isn't as great as one might think. According to the Bureau of Labor Statistics, the mean wage for a Physician with no seperate classification (dermatologist isn't classified seperately) is about $155K per year. http://www.bls.gov/oes/current/oes291069鈥?/a>





Another thing you might want to do is if you have a decent relationship with your family doctor or even if s/he seems like a nice person, call their office up and offer to take them to lunch to talk about their career choice. You can also bring up what they though of medical school the next time you go to the doctor. I asked my Primiary Care Physician when I was thinking about going to medical school. My doc is a DO and he said that if he were to do it all over again, he would have gone through a PA program rather than going through the entire medical program.





Best of luck to you!
1st you have to know that after you get your bachelors degree which will take you 4 years you have about 4 MORE years of medical school to do. AND then you have about 2 more years of school to do if you specialize in a different field of medicine.





But if you really want to do this i would major in biology or chemistry. Your university might tell you to go pre-med though
I just found out that to be in any medical position you can major in anything as long as you have the classes that are required to get into Med School. You can look up medical school requirement (heres a link to one http://www.studentdoc.com/medical-school鈥?/a> and it'll give you a list or if you have a school in mind you can go to the schools website and check their requirement. As for how much they make, I'm not sure,
if you are gonna be one u need to pay a tenion in your classes cuse some arent good like my i want a new on but is a dermatolgist a skin doctor(idk im only 13) i have one
Medication, and biology.





dermatologist make a good amount for a treatment.
Pre-Med, lots of money. You can look up estimated salaries for your area on the US Department of Labor website.
you should first major in the sciences.
biology
Pre Med
  • eyeshadow
  • Can anyone recommend a London based dermatologist specializing in black skin?

    I have been living in London for two years now and I'm having the hardest time finding a dermatologist or aesthetician with experience caring for black skin鈥攎y main concern is hyperpigmentation. I live in Central London so doctors in Zones 1-3 would be ideal.Can anyone recommend a London based dermatologist specializing in black skin?
    I'd recommend you try posting your question at the site below.





    I also found this post at another site:





    'Try the Laser Treatment clinic they specialize in Black skin. Their number is 08451 30 35 36 or 0207 307 8747. They are based at Harley Street London. And they are also relitively cheap.'





    HTH : )Can anyone recommend a London based dermatologist specializing in black skin?
    sorry i sure dont

    Is it common for a dermatologist to not be available after hours?

    I was being treated for a staph infection above my eyelid and was on my last day of antibiotics (on a thursday) so I called her office and they said she was out until Monday. The infection was still present and I wasn't sure if I needed to continue on antibiotics or what.





    The receptionist said she didn't take calls after hours because she was a dermatologist and her patients didn't have emergencies so no one was on call for her either.





    Should I find a new doctor?Is it common for a dermatologist to not be available after hours?
    Antibiotics are still working in the body even after you've completed the medication; has the condition at least improved?





    The receptionist is correct, dermatologic conditions are not emergencies that require immediate care. If you are that concerned, go to the Emergency Room or call your Ophthalmologist.








    鈥?ADDENDUM 鈥?br>




    My son had cellulitis near his eye, and our primary Dr. gave him an antibiotic shot in his @$$, plus a prescription for oral antibiotics; and then referred us to our ophthalmologist for follow-up. I still don't understand why you are seeing a dermatologist for an eye infection.





    Even for plastic surgery, the Drs. are on-call for emergencies such as facial injuries that require stitching or reconstruction. After surgery, you should be on antibiotics as a preventative; and again, it is unlikely that you would have a life-or-death emergency following that. The same for skin cancer -- it is an ongoing treatment that takes time.





    If you really feel that your condition will result in loss of the eye or your life before Monday, then go to the Emergency Room and have them call your Dermatologist.Is it common for a dermatologist to not be available after hours?
    I see (pun intended - LOL!); and again, if your infection crossed to the bloodstream, it would no longer be a dermatologic issue and she'd refer you to the E.R. for prompt treatment. I hope you are recovering nicely - take care! Report Abuse

    I would find a different doctor, one that does have patients with emergencies so she is available at off hours.
    IT'S OUR FATE TO WAIT FOR THESE doctors
    I would find a new doctor.
    yaa u have too

    I just went to the dermatologist today and I have enlarged oil glands on my nose?

    They basically look like flesh colored pimples. I got a smooth laser treatment, i saw some improvement, but they didn't go away. Is there ANYTHING i can do to fix these enlarged oil glands? they are so embarassingI just went to the dermatologist today and I have enlarged oil glands on my nose?
    What the skin needs is a product that clears pores and kills bacteria, thus preventing hair follicles from clogging and turning into pimples. This is what you should be looking for if you want to get rid of those ugly pimples.


    Before trying any treatment, I suggest you read this informational source that listed the most effective acne treatments available:





    http://www.17acnetreatments.com





    P.S. I also know someone who used to suffer from severe acne for 14 YEARS and finally got rid of it completely. She's sharing her story at her site:





    http://20six.co.uk/my-acne-solution





    Maybe you could learn from her inspirational story


    .








    Good luck.

    Anyone know of a good dermatologist's in Lancashire, england?

    I've suffered with acne for the last 8 years, or so. Antiobiotic's seem a waste of time, and would like to see a specialist for more advanced treatment. Anyone recommend anywhere, please?Anyone know of a good dermatologist's in Lancashire, england?
    Your nearest large hospital should have dermatologists. You need to see your G.P., and ask to be referred to someone.





    A dermatologist may decide to treat you with a course of ROACCUTANE, which gives marvellous results. If your acne is VERY bad, or hasn't improved with the antibiotic treatments, (which usually take 3 months or longer to work), you may well be prescibed it.





    GOOD LUCK

    How do I get a dermatologist?

    My acne is getting really bad and I want to get help. The first time, my doctor referred me to a dermatologist and he just prescribed some acne lotion for my nose and said to come back within 3 months if it doesn't work. I didn't go back. Now do I have to go back to my family physician and get another referral or what? What should I do?How do I get a dermatologist?
    Hey,





    When i was younger i had really bad acne. I went to the doctors the same as you and they prescribed me some topical lotion the same as you and told me to go back if it didnt help. I went back, and they then put me on some antibiotics which really helped for a while.





    If you have already had a referral then you shouldnt need to go back to your GP, so call the hospital directly *dermatology department) and say that you want to book another appointment. Once youve had the initial referral you dont need another one....





    Go back to the dermatologist and say that it hasnt worked, and that its really affecting you. Anti-biotics might be the next step for you, but it really clears your skin and you'll feel so much better.





    Good Luck !

    How do I become a dermatologist?

    My highest level of education is high school -- where do I go from here?How do I become a dermatologist?
    Dermatologists are Doctors of Dermatology. It is a specialty of medicine. Start by checking out pre-med programs.





    Dermatologists are physicians who diagnose and treat disorders of the skin. Dermatologists may be specialists who treat diseases and tumors of the skin, hair, nails, and sweat glands. Dermatologists may also specialize in dermasurgery and practice surgery to treat skin conditions. All dermatologists must rely on advanced skills and exercise critical judgment in their field.





    Prerequisites for medical school require two to four years of undergraduate training in a science or pre-medical degree. Admissions also require successful completion of the Medical College Admission Test (MCAT). Medical schools also consider extracurricular activities, grade point average, and letters of recommendation. Various schools and colleges offer flexible course schedules that range from day, night and weekend classes and training sessions.








    How do I become a dermatologist?
    You have to go to a university which offers medical studies.Becoming a dermatologist requires almost the same courses you have to take in medical school as becoming a medical doctor. After graduation, you need to put in time for residency at a hospital which has a practice of dermatology. In total, it takes about 6 years minimum.


    Go for it, aim high, study hard and become a doctor.


    Good luck!
    My dear Mom





    Due to the time difference, I just saw your request (it's 4:30 PM over here)...so that's why I am a little ';late'; in answering. But I saw the above answers and the 2 guys (tom and Raul) gave you the correct instructions, honey.





    You will have a total of 12 years ahead of you !!!! Are you really ';prepared'; for this??? I mean, I acknowledge your aspiration, but with the baby, your housework, etc. ---it will be HARD---for you, sweetie!!!





    I went several years to College (when my son was in Kindergarten) just for the fun of acquiring knowledge and those were some great years....But: be aware, it's going to be rough with a small child to care for. Usually people do it ';the other way around';...They go to a College right after high school, get their education...and then...they ';start a family';...





    Besides, studying Medicine is one of the most difficult ';subjects'; you can choose, so be aware of a whole LOT of studying!!! But nevertheless, I sure wish to encourage you and wish you all the strength you need ! My prayers are with you ! Send you my best greetings along with all my love, care %26amp; concern...your friend...Annette***
    Next step you need to get an associates in arts and sciences. (2 year college) then University - make sure you take the classes you'll need for that profession. You'll need to choose major and minor studies. You can go to your community college and schedule an appointment with one of the counselors there. Its free. They can give you the whole rundown of what you need. GOOD LUCK!
    my dad is a dermatologist. he said.. go to good collage and do well, doing community service helps lots. you go to your university for 4 years, then medical school for 4 years, then dermatologist training for another 4 years. then you can get a job anywhere.
    A dermatologist is a physician, with a specialty. You need to get a regular medical education, and then spend a specific number of additional years to prepare for a specialty.
    Plan to go to college for a science degree


    then on to graduate school to get a masters


    and then a medical school for an M.D


    a doctorate





    you will have a few years of hard studying





    all the best
    You can start by taking general college courses. I'm sure you'll probably need biology as well as other medical sciences.
    go to college. major in a scientific field like biology or chemistry, etc. and then you will need to go to medical school after college.
    if your dead set on it you can take medical courses from a university then move to a medical school
    College then medical school then residency.
    Buy a box of Derma's and just practice till youve got it down pat. Then youll have to go to Dermacollege for 3 years its easy.
    do a science degree in university





    go to medical school





    do a medical residency in dermatology
    Start with a local college. Get your basics in then speak to the counselors and ask them where to go from there.
    Go to college and see what field of education you need to take.


    get a degree


    and apply for the job





    simple
    college...medical school...specialize in dermatology...
    college for dermatologist
    to some kind of university that you can study medicine- dermotology
    Thats going to require a medical degree, you should look up some scholarship programs online.
  • eyeshadow
  • If i go to a dermatologist will they automatically prescribe me accutane?

    i really want to go on accutane, my skin has always been not too bad, nothing i thought i had to worry about but now, its worse and im embarassed to leave my house, but some people would say its not too severe... so im wondering if i go to a dermatologist, which would be my first time, will they automatically prescribe me accutane, or will it be hard to convince them? i really dont want any creams or anything because i cant really wear them during the day because of my makeup... If i go to a dermatologist will they automatically prescribe me accutane?
    Creams and various other oral meds should be tried first. Creams are usually put on before bed so shouldn't cause a problem with makeup. Please put off accutane as long as you can the side affects can be devastating. While it works for some people it can trigger depression in others. That's what happened to my nephew and unfortunately he killed himself. Make sure you are aware of all side affects of any meds they put you on it could save your life.If i go to a dermatologist will they automatically prescribe me accutane?
    ok, to take accutane is a very serious this, it's not just like okay try this. derms will only prescribe this if they only feel it's necessary b/c the side effects are very VERY serious. it sounds like you really don't have that bad of acne, you admitted it yourself. there are creams out there where you can wear make up. there are actually several. accutane takes forever to work and a lot to do to take it. i am assuming your a girl (your pic) you have to get blood work, take 2 pregnancy tests (can cause severe fetal damage). most derms are very hesitant to start pt's on accutane b/c it could possibly ruin their practice, if someone hurt themselves on it.
    More than likely they wont give it to you right away and if you have never tried anything else before, i suggest trying something not as severe. I was on accutane and had to convince my dermatologist after trying many other things. Accutane can be complicated and has many side effects. I did fine on it besides the fact that i had really chapped lips, but other people werent as lucky. I suggest trying a few other things first, especially if your acne isnt that bad. If you have any other questions about it let me know. Good luck!
    you won't be able to convince them


    it would be against the law for them


    accutane kills your liver


    so check out the side effects


    and get more information before


    you go see a dermatologist

    What do i major if i want to be a dermatologist?

    okay i'm in the 10th grade and my best friend mom is a dermatologist. Her mom has so much money and my friend gets anything she want...sometimes when i go shopping with my friend she buys me a couple of outfits...And OMG you should see there house...what do i major in if i want to be a dermatologist....so i can have money like that?What do i major if i want to be a dermatologist?
    premedWhat do i major if i want to be a dermatologist?
    Pre-Med: first:


    Medical school next:


    internship at hospital next


    residence at hospital next


    specialist training next:


    .


    You're looking at somewhere around ten years of medical school and your specialist training.


    .


    It is not easy becoming a medical doctor:
    First, Go to college and complete a premed curriculum in any major.





    Second, Go to medical school. Any American medical school is great.





    Third, Complete a residency in dermatology.
    check on http://www.ehow.com/
    You can major in anything you want as long as you take the premed courses in bio, chem, physics, math, and English. Keep your college GPA well over 3.5 and do well on the MCAT (med school entrance exam). Then you've got 4 more years of medical school followed by 5 years of residency (training) before you can get a job on your own. And if you're just in it for the money, you're not going to make it too far. It's a ton of work.
    You shouldn't go into a job just because you like money. You should go into a job you enjoy. Don't get stuck in something that you don't want to do.


    But anyways:


    For dermatology, you need a 4-year college degree (major in anything you want/can, and take the pre-med courses in bio, chem, physics and math), 4 years of medical school (that's where you start to specialize in dermatology or anything else) and then a dermatology residency (at least 3 years).





    You need to become a medical doctor or a D.O. to become a dermatologist.





    4 years of undergrad study


    4 years of medical school


    1 year internship


    3 years of internal medicine residency


    2 years derm fellow= 14 years training/schooling.





    Along with the MCATs, USMLEs, and Board certification.

    How do I become a Dermatologist?

    I'm 17 and I really want to become a Dermatologist. I haven't done too good in high school but I'm definitely going to do better my senior year(which is this year). I want to go to a community college for a year or two(in Florida) since it's cheaper and then apply to a UF, UM, or FSU. Is it bad to go to a community college first? What should I do? And What classes should I take?How do I become a Dermatologist?
    texier,





    You need to be careful on who you listen to on this website.





    People can give you bad advice. Like the one previously before me, she said that you should major in pre-med. Well, to tell you what, there is NO SUCH thing as a pre-med major. Pre-med is more of a route or journey you take in order to get into med school. This route consists of class requirements such as





    1 year of biology





    1 year of general chemistry





    1 year of organic chemistry





    1 year of physics





    1 year of english





    1 semester of calculus





    and other humanities courses depending on the med school of your choice.





    Look, high school doesn't really matter much when you are applying to med school.





    What matters is what you will do in college. You'll need to gain experience in the medical field by either volunteeering or working in a medical setting. This way you can write in your personal statement on why you want to be a doctor so bad.





    As for after high school, I recommend that you go straight to university. JC is a waste of time and you may not be able to get your classes. Also, it feels too much like the 13th grade instead of college.





    If you have any more questions, feel free to email me.





    http://www.pre-med.infoHow do I become a Dermatologist?
    Med school....... good luck. You need really good grades and lots of $
    You have to major in pre-med and then go to medical school. Dermatologists are medical doctors.

    How do I get my doctor to refer me to a dermatologist? are reluctant but it does not cost them anything.?

    I am 22 and had not got spots regularly for a few years now but recently I have started getting them again. I get very big spots on my cheeks in symmetry, I get one big red spot on one cheek a few days later it will come on the exact same place on the other side, they are really and leave big red marks, it took years for other marks from this condiction to fade a little bit, but don't want adult acne heard the marks for this are really bad with no fading etc.





    Does it cost the surgery? I heard they can't be bothered filling paperwork in. Also I personally haven't been yet am expecting a no because a friend had very bad spots and they tried to talk her out of it.





    It's not normal for people my age to start getting red spots that are very big, that sting a bit and are infectious, it's not an allergic reaction or anything in case you are wondering I always had spots like these in the past but I though I had seen the last of spots, they are bigger than my head.How do I get my doctor to refer me to a dermatologist? are reluctant but it does not cost them anything.?
    if your gp will not help you, find another doctor who will... sometimes doctors at the walk-in-clinics care more for their temporary patients than the person's gp..How do I get my doctor to refer me to a dermatologist? are reluctant but it does not cost them anything.?
    you'll probably grow out of that. A doctor doesn't have to ';refer'; you to dermatologist...just look in the yellow pages of your phone book and choose one that's near you, set an appointment and get an opinion.
    you make no sense. talk to your doctor.
    are we talking large acne? very painful and infected? sounds like cystic acne. you really don't need a dermatologist for that, especially if it's only a few. you're still very young. it could be hormonal. i get them when my period is due and i'm 46. by the way, what surgery are you talking about?
    Cry when you talk to him. Tell him the spots are ruining your life. Change doctors if he won't help you.
    Maybe you are expecting resistance where it will not come. Just because your friend had a problem, it does not mean you will. Explain your concern to your doctor, pure and simple.





    The doctor will listen and respect you if you are honest and if you also listen. Do not be demanding, just open.





    Good luck with this!

    Dermatology - What other types of physicians might work with a dermatologist?

    What other health care professionals may work with a dermatologist and the reasons for the connection? For example, orthopedic surgeons may refer their patients to physical therapists for rehabilitation.Dermatology - What other types of physicians might work with a dermatologist?
    Plastic Surgeon


    To remove skincancers





    Pathologist


    To check histology of lesions





    Radiotherapist


    To irradiate certain skincancers





    Immunologists


    To assess whether skinlesion is related to systemic (immunity) disorder





    etcDermatology - What other types of physicians might work with a dermatologist?
    A Dermatologist may refer to an Oncologist for someone with skin cancer.

    How do I get rid of milia under my eye without having to go to the dermatologist?

    123456789,


    The key to getting rid of milia is realizing that they have no escape route, those little bumps are trapped under the skin. So, to get them out, you鈥檒l need to have a professional like a dermatologist extract them. You can extract the milia yourself, but this involves risks. You need to ensure that your milia are not symptoms of some underlying disease or illness. Also, if you extract the milia yourself, you may have trouble completely pulling out the cysts, as the removable process may prove too painful. Otherwise, if you are confident with your health and courage, you can cleanse your hands and face. Then wet a cloth with warm water and apply it to your face for a few minutes. Gently apply a sterile needle to the centre of the little white bump to create a tiny opening in the skin. Wrap your thumbs in a clean tissue and, using your thumbs, proceed to gently squeeze the contents of the little white bump out. Finally, cleanse the area of the extracted little white bump with an astringent. In cases when detailed medical information is required you would be advised to contact your doctor or dermatologist.


    Hope this helps


    Matador 89How do I get rid of milia under my eye without having to go to the dermatologist?
    You can check with a facialist who does extractions but most of them won't touch milia close to the eyes.





    The other option is try using exfoliants regularly on the area to try and get rid of them that way. Maybe something like Dermologica Microfoliant which is a chemical exfoliator and fairly gentle for around the eye area.
  • eyeshadow
  • Where is a good dermatologist or a facial clinic in Metro Manila?

    Help! Zits have invaded my face again! Which clinic or dermatologist could you recommend, preferably somewhere in QC? I have gone to Clarity once but I didn't like the experience. After they injected the pimples, I got scars for two whole weeks. Everybody else I know didn't get those dark scars when they went to a derma.Where is a good dermatologist or a facial clinic in Metro Manila?
    tight budget and good facial center don't go together. whatever you do, don't fall for the ';facial center'; your neighborhood parlor or carinderia added to their name. go for the big ones like belo or calayan. i tried a cheaper facial center before..the services are cheaper but they pester you to buy their super expensive medications because according to them, the treatment you just had with them won't work without the medications. worse, nothing worked. so they ask you to come again and go through it...again. *sigh. talk to belo or calayan.Where is a good dermatologist or a facial clinic in Metro Manila?
    Belo Medical Center.....................I heard good stuffs about them............
    Belo Medical Group..
    I got a Facial at Belo before.. and it hurts a lot... I never know facials are supposed to hurt. Ok lang yon.. Ksi.. ang gwapo ko naman..
    Vicky Belo Clinic
    tirisin mo na lang at kuskusin mo ng kalamansi





    1-800-game-ho-bini
    try Calayan or Vicky Belo. they have clinics somewhere in QC. check their websites for complete address. good luck!
    http://www.geocities.com/manila_hospital鈥?/a>
    Derma Aesthetika


    Dr. Geraldine Emperador


    3/F Unit 9 Body Senses


    Robinsons Galleria


    Ortigas Ave., Quezon City


    Phone 634-2047


    --


    Surgiderm


    Dr. Roberto Cuerto


    Unit Y, Bodyplace, Level III, Robinson's Galleria


    Phone 633-5610


    Easycall 141-983038


    --


    Via Dermatologica Clinic


    Dr. Liza Fernandez


    Dr. Nelson Abelino


    Unit H, Bodyplace, Level III, Ortigas Center


    Quezon City


    Phone 636-4916
    vicky belo. or doctor calayan
    tight budget and quality dermatological service still goes together! try dermstrata. it is located all across the metro. you can also find ads about that clinic on newspapers. you can find it on all ( i mean all!) major malls. regular facial costs around 400 pesos. you can also try dermline. costs around 400 too. but i prefer dermstrata better. their services are much more detailed.
    Forever Flawless or Belo medical center





    here's the webiste there's an address for QC clinic.





    http://www.belomed.com/clinic.htm





    ****


    ok, forever flawless is less expensive that belo clinic. but they still do a good job and flawless is for the masses. There's one in megamall.
    There was a skin care clinic in Pedro Gil. I used to have my facial treatment there. it will cost you but not too much. I enjoyed thier service alot. I forgot the name of the clinic. If you are in Taft ave. it is at the bak of Gasoline station and beside 7-11 going to St. Paul Manila.

    How many years does it take to become a doctor, dermatologist, and a dentist in u.s. ?

    To become any of the things in your list, you first have to get a Bachelor's degree and take the necassary requirements for dental or medical school. To get a Bachelor's degree takes 4 years.





    For medical school, it takes another 4 years. Once you finish medical school, you can specialize. You do this during residency. For dermatology, I believe it is a 3 year residency.


    All Doctors are required to do some form of residency and it can last anywhere from 3-8 years depending on the speciality..





    Dental school takes 4 years as well and also requires a Bachelor's degree to attend.





    So the short answer is





    8 years for a dentist





    11 years for a dermatologist/doctor.How many years does it take to become a doctor, dermatologist, and a dentist in u.s. ?
    A doctor is 8 to 11 years

    How much does a dermatologist visit cost? Numbers only please?

    I need to know how much it costed you or someone you know to visit the dematologist for a bad case of acne. I just want numbers though so I know how much I should be saving up for to get it done. No insuranceHow much does a dermatologist visit cost? Numbers only please?
    Non insurance office rates are different from doctor to doctor, from area to area.


    Call your doctor's office and ask how much it will be WITHOUT insurance. It's the only way to know.

    I have very fair skin which turns red way too easily.. is there something a dermatologist can do?

    I use some acne medications which may be the reason, but even if I stop using them my face still ill say blushes easily. Iduno if roseacea could be what it is. Like I won't be embarrased but ill still turn red, and when Im active like with running still the same issue..


    would a dermatologist be able to give me anything to help with this?I have very fair skin which turns red way too easily.. is there something a dermatologist can do?
    I am very fair too, and that is why its so noticeable. Unless its rash like and red, then no, not much you can do about it. I turn bright red when i go running, no more than anyone else, but because I am so fair, you can notice it more than my BFF who is super tan.I have very fair skin which turns red way too easily.. is there something a dermatologist can do?
    I've been using Cindy Crawfords Meaningful beauty products and I hve roseacea, ever so slightly. On me it looks more like blush. But either way this stuff rocks and even my husband said my skin looks more even and less red. It seems to work and it costs me 39 a month for the face wash, lotion, evening lotion, mask, and eye creame. YOu can't beat that.
    Yes a dermatologist would be able to help.


    But, also just so you know, there are Aveeno lotions and face washes that can help. They even out skin tones and help with redness in the skin.
    I have fair skin also, and I had the same problem. I also began to develop some ugly ';pimples';, and I am 56 years old.





    I went to a dermatologist and he said I have roseacea. He initially prescribed antibiotics for the pimples, and metronidazole cream to be applied twice a day.





    The pimples have gone and I still apply the cream. The cream has kept the pimples at bay. My face does still tend to get red. To combat the redness, avoid alcohol, spicy foods, and too much sunlight.





    A dermatologist can definitely help. Please see one. Left untreated, roseacea can lead to broken blood vessels in the nose and the development of a nose like W.C. Fields (I hope you are old enough to know who he is, if not, find a picture of him on line).

    What would I need to do to become a dermatologist?

    Meaning what schooling would I need and stuff? do you think its a wise career choice? thanksWhat would I need to do to become a dermatologist?
    it's a long road, but very doable. Get good grades in High School, take all your premed classes and do fairly well (inorganic chem, organic chem, physics, bio, plus your major,) take a leadership role in a volunteer cause or group you believe in (so you have something good on your transcript and something to write your essays about,) take the MCAT's and score pretty good (don't need to be great, just pretty good,) get some med school interviews, get into a med school, take out MONSTROUS student loans for med school, study whenever you're not doing something else for the next two years, pass part one of the boards, then the second two years of med school are clinicals, so long hours but easier study time. then pass part two of boards, graduate from med school, interview and match at a derm residency (I think that's four years?)





    that's it! sounds like a lot, but if you take it one little step at a time, one milestone at a time, it's doable. But, here's the rub. You really, really gotta want it. The only people who don't make it thru med school are the ones who weren't there for the right reasons. They were the ones who went because it's what their parents wanted, or they just wanted to see if they could get in. If you have a burning desire to do it, you can. If not, it would probably not seem worth it.





    good luck!What would I need to do to become a dermatologist?
    Go to school
    Get a pre-med 4 year degree, then apply to medical school. Good luck.

    Does medical insurance cover the cost or some of the cost to go see a dermatologist if you are 31 years old?

    I have a little bit of Acne scaring and want to have it taken care of, i was told that if you're past the age of 23 it doesn't. Is this true? will i have to pay for the whole thing because I'm 31 and not under the age of 23 ? please i need people that know a definite answer no I guesses, or I thinks.Does medical insurance cover the cost or some of the cost to go see a dermatologist if you are 31 years old?
    ~~I think what you heard about is the prescription Retinal-A. This is not covered by insurance after the age of 23 because it is also used for removing wrinkles, so then gets considered as cosmetic.





    If your medical insurance covers dermatologists (most all do, but always check with your insurance company, then you will also know if you need a referral, which providers you have to choose from, etc), prior to setting an appointment with the doctor. If there is a medical reason for any procedure or prescription to be necessary, he can get authorization by justifying the need to your ins. co.





    If it is strictly cosmetic and you have coverage for a dermatologist under your plan, then the visit will be covered, but no procedures or RX's will be.~~Does medical insurance cover the cost or some of the cost to go see a dermatologist if you are 31 years old?
    If your medical insurance does not cover the cosmetic procedure you may want to consider a Discount Medical Plan. There are no such restrictions or limits, because the plans are not insurance.
    It has nothing to do with your age.





    It's all about, is it medically necessary, or is it cosmetic. What is it you want them to do? Fix the scarring? That's likely to be cosmetic.





    You will need to call the number on the back of your card and ask.
    It shouldn't have anything to do with your age. It just has to do with what type of coverage your policy provides.





    There should be a number on the back of your card that you can call to ask questions. They'd be much more likely to give you an accurate answer than we would.
    healthplans.my-age.net - here is my health insurance plan. As I remember they can provide such a service.
  • eyeshadow
  • Facial mole removed by a plastic surgeon or dermatologist?

    My facial mole is about 1/2 centimeter in diameter and it's located at the bridge of my nose near my eye. A plastic surgeon charges me $350 to get it removed and a dermatologist charges $200. I heard a plastic surgeon is preferred when getting a facial mole removed because they are trained and have the experience to give you a result with minimal scarring. What do you guys suggest I should do?Facial mole removed by a plastic surgeon or dermatologist?
    I work in surgery and I've seen many nice wound closures by doctors other than plastic surgeons. As a matter of fact, my dermatologist removed a growth from my back in February and the scar is not visible. If the mole is small enough that your dermatologist can do it in the office I would consider it.Facial mole removed by a plastic surgeon or dermatologist?
    That's a tough one I think I'd stick to the dermatologist because it's a mole and he's likely removed more then the plastic surgeon - I am pretty sure he will ensure you won't scar as well as the other doctor - and if he felt it was out of his 'hands' with a scar he may send you to the surgeon anyway.





    If it's raised it will be biopsied anyway to ensure it's just a simple mole - I have had a lot of moles removed by a general MD and Dermatologist. I'm still here to give advice.
    i agree go to a plastic surgeon its better to be safe than sorry
    i had one removed by my dermatologist and i had no scar left...
    dermatologist

    Has any datrk skinned person been to the dermatologist for dark scars?

    if yes - did what they prescribe work to lighten them?


    what did they prescribe?Has any datrk skinned person been to the dermatologist for dark scars?
    hydroquinone.... :o)

    How much does it cost to visit a dermatologist in melbourne australia.?

    Just a guide or ballpark figure would be good.How much does it cost to visit a dermatologist in melbourne australia.?
    my workmate went to a dermotoligist for about 20 mins and it cost her about $160 .....that was in Sydney, but im pretty sure they would all charge similar sort of rates ...

    Where can I get clobetasol proportionate without a prescription? cant afford 2 go see dermatologist again?

    I keep get pimples in my scalp, or hair bumps i dunno. anyway the dermatologist prescriped me doxycycline and clobetasol proportionate. it worked at first, but now my scalp is starting to break out again. i can't afford to go see another dermatologist so how can i get more of the stuff? can a pharmacist prescribe it to me?Where can I get clobetasol proportionate without a prescription? cant afford 2 go see dermatologist again?
    Go to your local GP and he or she may prescribe something for you they bulk bill and put it through medicare well most GPs do anywayWhere can I get clobetasol proportionate without a prescription? cant afford 2 go see dermatologist again?
    I've been in that situation and there is really nothing you can do about it. There are things you can do to try to reduce the blemishes and get rid of acne/pimples temporarily but you will always be finding yourself cleaning your face or your body depending on where it's at.





    I mean of course your acne should go away eventually but for some this is the case their whole life.





    I'm 29 years old and have been suffering from these little pests since I was a teenager. I tried all sorts of face washes from the local grocery store and what not, but nothing seemed to work. If anything it would just make my skin dry and itchy.





    Literally the best acne face wash I've ever used has been this stuff called clearpores. It has got rid of the acne to quickest and has been a success ever since I got it. It's unique cause of it's herbal supplement you take a long with it that helps your skin from the inside out. I saved money on it at skincleansingsystem.com at the time which was helpful.





    You should really look into this stuff and if you can to assure the best results, get the 3 month supply because if it doesn't work it will still save you butt on the return policy.





    Well good luck and best wishes.

    How long does it take to become a dermatologist?

    I'm thinking about itHow long does it take to become a dermatologist?
    4 years undergrad


    4 years medschool


    2 years intern with a specialty in derm


    2 years residency





    12 years totalHow long does it take to become a dermatologist?
    about 10 years. you go to med school for 4, rotate for 3, and intern for 3
    It requires gonig to med school.


    So, four years basic college + four for med school.

    I am planning to go see a dermatologist about my acne what should i be expecting?

    what kind of medication will they give me???


    i have waited this long to go because im embarrassed but i cant take it anymore ive been depressed for way to long. what do you guys recommend to do??


    thanks to allI am planning to go see a dermatologist about my acne what should i be expecting?
    hope this helps





    http://www.skincarephysicians.com/acnene鈥?/a>I am planning to go see a dermatologist about my acne what should i be expecting?
    always try natural methods before doing that. i wud suggest u 2 try expertpress.org/acne
  • eyeshadow
  • Can your family doctor instead of a dermatologist write prescriptions for Accutane or Differen gel for acne?

    yes , usually the family doctor does do this . you should occasionally see the specialist for the current / advise on what drug / medicine is best for whatever your condition is . medicines can change newer things might become available . after you know what is best , you can fill it and then you then just bring the tube or bottle with you to the family doctor office because the family doctor needs to see what is the exact size / color / brand / name of medicine that you want .





    the family doctor is a good person to help manage the various prescriptions a person may have . i like my internal family doctor to help weigh in on decisions and to make sure that all the things are coordinated / work well for my system. the family doctor is often more available for appointments don't have to wait months to see . god forbid you should have to cancel , you have a huge fee / and wait six more months for another perfect window.





    i think specialists might be the right people to make selections / about what medicine is best for whatever the situation , but the family doctor can execute that doctors advise . doctors do coordinate together if any questions , it's helpful and makes me more comfortable. my family doctor is quite helpful in explaining more to educate/ teach me / whyCan your family doctor instead of a dermatologist write prescriptions for Accutane or Differen gel for acne?
    Yes. However, if he or she feels that a dermatologist would better be able to evaluate and treat your condition, you may get a referral.Can your family doctor instead of a dermatologist write prescriptions for Accutane or Differen gel for acne?
    any licensed physician may write for any approved drug. Question is, will they?
    No gel or any cream is really helping with acne. It just fills up the pores and you get more acne. The only treatment with thousands of proven results is Low Level Laser therapy, e.g. acne is treated with derma laser without scars and any side effects - the healing process is starting immediately. You should look for a laser specialist. It will help definitely!

    Do dermatologist give treatments for scars and razor bumps?

    Are they efficient? How about for excessive hair growth? No stupid answers please thank you all =}Do dermatologist give treatments for scars and razor bumps?
    There are creams so I believe there are treatments available for those cases, sorry I couldn't be more helpful but hope!

    How do I complain about the services I received from a so-called dermatologist?

    There must be some ';Board'; where malpractice can be reported to. This dr. paid no attention to my needs, she just glanced at my rash and told me it was atopic dermatitis and to change my soaps...prescribed me some topical ointment and sent me over to her ';in-house'; vitamin person who had a product line available immediately. I personally think she was more interested in selling her products than caring about the rash. PS...went for a 2nd opinion and was diagnosed with dermatomyositis which can be terminal if related to a cancer (of which I am...and the 1st derm...knew this)How do I complain about the services I received from a so-called dermatologist?
    this lst doctor you went to was she an M.D. board certified


    dermatologist......?





    It is obvious that atopic dermatitis and dermatomyositis are


    different skin disease......atopic dermatitis is usually caused


    by allergies and I believe dermatomyositis is an auto immune


    disease.





    Maybe you should get a third opinion by visiting with a


    rheumatologist or endocrinologist. good luckHow do I complain about the services I received from a so-called dermatologist?
    That's terrible, but not surprising! Contact the American Board of Dermatology
    Call the American Medical ASsociation or go online. If they get enough complaints or lawsuits they can be disbarred.

    Having dermatologist remove susipicious mole without inxurance?

    Im going to my dermatologist to get a biopsy. I will have health insurance in two months. If this turns out to be cancer, which obviously I hope it wont, will I not be able to get insurance? (its through my job) I will be getting Empire blue cross blue shield, should I pay for it for the next two months? Will this change anything? Also does anyone know how much a biopsy is without insurance?Having dermatologist remove susipicious mole without inxurance?
    Talk to your dermatologist. Mine had just gotten a new laser and was so excited to use it he didn't charge me anything, of course there was no expensive biopsy involved, but explain your situation and see what they say.

    Should i wear makeup to the dermatologist?

    i am having a followup in my acne, can i wear face makeup like concealer and powder?Should i wear makeup to the dermatologist?
    No, don't wear makeup to the dermatologist. They need to see how your skin looks, feels and seems. I say just carry the makeup in your purse and put it on in the car right after the appt.Should i wear makeup to the dermatologist?
    Hello....are you trying to fool the dermatologist???





    Go with a clean face, depending on what they tell you put your makeup on in the bathroom when you are done with the follow-up.
    no consealer or powder, just mascara, eyeliner at the most
    dont wear makeup to the dermatologist! try only wearing mineral makeup
    no don't wear any make up they need to see your skin
    probs best not to so they can see wot ur skin is actually like

    How do i get to a dermatologist?

    And is it free on the NHS? Will the doctor refer me if i ask him/her? I have lots of moles on my legs, arm neck and and back, even the hips, i want them checking to make sure they are not cancerous and make sure i'm safe.





    ThanksHow do i get to a dermatologist?
    Doctor should refer you! tell him/her about your concerns!How do i get to a dermatologist?
    Your GP has an obligation to refer you to clinical specialist, especially if you are extremely anxious about this. Say you want a second opinion. Some people have lots of moles which are harmless except when they begin changing. You can only have complete assurance that the tissue sample is cancerous or non-cancerous by pathology examination
    your gp will probably blow you off but you can ask, otherwise you can write to the dermatologist yourself, however, one really only checks moles out if they have changed shape or size or consistancy.. for example if you have had them there looking the same all your life, ts no big deal. just because you have a lot doesnt mean they are cancerous.
    You need to get referred by your GP for free NHS treatment or pay to go private.





    I needed to see one for dermatitis and was fobbed off and eventually changed doctor and had to keep going back before I got referred. It's they're job to help you. If they try dismissing you you may have to complain a bit or keep going back.
    your doctor can do this. If a mole bothers you they can remove it, if its worrying they can refer you to a hospital treatment ward.
    your insure.comp. will approve first..and or your doctor will sent you.. i have united health care,,i can to any doctor for any reason with any doctor sending me or what
    usually a referal from your family doctor.
  • eyeshadow
  • I want my acne gone for good. What should i tell my dermatologist?

    I've already gotten many prescriptions including pills-DORYX 150mg. the creams im currently using are differin and duac. My derm is prob happy that she can keep on making money off of me but all docs are like that,so what can i ask or do to finally make my acne go away after so long? My appointment is in 6 dayss. serious questionnnnnI want my acne gone for good. What should i tell my dermatologist?
    Make a list of all the medication you are/were on and give it to him/her. Describe you acne in detail, and if it's a good derm they will examine your face or wherever you have acne. Give each medication at least three months, sometimes when you start new medicine for acne it will make it much worse before it gets better. Drink lots of water, change your pillowcase and DO NOT touch your face!I want my acne gone for good. What should i tell my dermatologist?
    You should tell him '; I want my acne gone for good';
    TELL her to put you on ACCUTANE NOW! I once had a derm that only cared about the money. Then i switched to a nice guy who actually cared about helping his patients and helping them get clear. Get accutane or switch doctors, don't be a personal piggy bank to anyone.
    Hi, Mish


    The effectiveness of safe Acne Treatments has been studied extensively by many international organizations and research institutes. Unfortunately, this research is often not published in a suitable way for consumers to be able to compare the different Acne Treatments available on the market.





    In fact, the lack of source citations on the Internet is a little bit scary. There is a system called:';Acne Free In 3 Days';. You will be pleasantly surprised by the book. All the sources are cited and he even has an extensive bibliography in the back.





    Why should this matter for you?





    For starters, it meanst the information in the book is accurate and you won鈥檛 have trouble following the things it has to say. It is very impressive that it not only gives you a detailed account of the things against acne, it also has the research to back it up.





    The writing style is also easy to read. You can feel and understand the pain becasue it鈥檚 something you have felt as well.





    This isn鈥檛 just another marketer trying to sell you something, This book is truly something which is there and cares about the results you get.





    You can read more information about this book at the link below:

    Are LPN nurses needed in a dermatologist office?

    Im a nurse at a pediatrics office and would much rather be working with a cosmetic plastic surgeon or at a dermatologist office. But, im not sure LPNs are needed in that kind of office setting. Any information?Are LPN nurses needed in a dermatologist office?
    Go to a dermatologist's office and ask!

    Can i still see a dermatologist without health insurance?

    i have some blemishes on my face so i want to go to a dermatologist, but do they all require insurance?how much would it cost for an appointment and would they beable to give me something over the counter so i dont need a prescription?Can i still see a dermatologist without health insurance?
    You could call the doctor's office and ask about their payment methods. For example, ask if they require payment at the time of treatment, or do they bill? They will still see you without insurance, but some doctors expect to be paid right away, so make sure you ask abut their treatment and visit fees before making the appointment.

    Should i give up the dermatologist cleanser and go back to proactiv?

    i had been using proactiv for a while but there's still some acne on my chin that don't seem to clear up, i went to a dermatologist and he gave me a set of cleanser, toning pad and lotion and moisturizer. i've been using them but been seeing one or two pimple breakout everyday.





    since i already got the prescription he got me, aczone(dapsone 5%) cream, should i just go back to using proactiv and use the cream for spot treatment?Should i give up the dermatologist cleanser and go back to proactiv?
    don't use either one


    if there are still problems when you use the proactiv or the stuff from the dermatologist then...start using something different


    [Pro-Sarcasm鈩

    Dermatologist in KY or OH that accepts medicaid?

    My daughter is breaking out with these white blotches and i know if i take her to a dermatologist and pay out of pocket it's going to be outrageous. Any idea's?Dermatologist in KY or OH that accepts medicaid?
    There is a new product out now that is helping this, You can go to


    { genniesalve.com } and check it out . Good Luck

    How should I approach my dermatologist about Accutane?

    I have had problems with cystic acne for about 5 yrs (since age 22). I finally got an appt. with a dermatologist. It seemed to me like I would be a good candidate for Accutane. But the derm. couldn't see me for 5 months.





    In the meantime I started using benzoyl peroxide, not as directed, but really slathered it on and followed with lots of moisturizer. It has made a big difference. I'm afraid now I might not get Accutane, because I don't *look* like I need it, but I do!! I don't want to keep using the bp forever, because even with moisturizer it pulls the skin tight, and makes me look wrinkly when I smile.





    I know I should have documented with photos, but I couldn't bring myself to do it. I don't want to go thru lots of antibiotics and creams and break out again before I get to try Accutane. Any suggestions?How should I approach my dermatologist about Accutane?
    a dermatologist will not prescribe you accutane untill after she prescribes you every other medication and it doesnt work. You really should of went to the dermatologist when you first started having a skin problem, 5 years ago. Then you could of tried all the other medications they are going to prescribe to you first and been through all of that already. It is very fair that they will prescribe you for accutane, untill you have tried every thing else and it doesnt work. Accutane comes with alot of responsiblity and directions. They try not to put many people on it. especially females, because if you get pregnant while on accutane or even a month after you stop using accutane, you will be basically forced to end the pregnancy because accutane causes serve problems for babies. Also, a dermatologist will not prescribe accutane during the summer months. They like to start around Oct. because accutane drys your skin soo badly, that the summer heat makes it worst.

    How much does one facial microdermabrasion procedure cost from a decent dermatologist?

    to be honest, i dont know what that is. but i'm sure its not cheap if its a big procedure at a dermatologist. good luck%26lt;3

    Would it be a waste of money if I saw a dermatologist?

    I am 12 (borrowing my sister's account).


    I get all these zits all over my face, and it seems like my facial cleanser isn't working.Also, all up and down my arms I have these little bumps. They don't itch or hurt or anything, but they look so UNATTRACTIVE.


    Do you think a dermatologist could help me? Am I too young to see one?Would it be a waste of money if I saw a dermatologist?
    There's no age restriction on seeing a dermatologist, but I would say it would only be worth your money if you have SEVERE acne. The creams, lotions, and treatments that they prescribe can be very expensive (any where from $45-$150 a month!) and generally aren't covered by insurance companies. If your acne is more of a mild case, I wouldn't recommend seeing a dermatologist quite yet. You're still pretty young, so I would recommend shopping around for the right cleanser. Everyone's skin is different, so it may take you a little while to find what skin care routine works for you. Good luck sweetie!Would it be a waste of money if I saw a dermatologist?
    Perhaps I'm the wrong one to be answering this quesion, but as a dermatologist, I'd have to say that if you have a skin problem that is causing you trouble, then a good dermatologist should be able to diagnose you and treat you. There is no age limit to seeing one of us, as we actually see newborns, too. Good luck.
    At your age it is quite familiar to hear about girls and guys suffer from all these pimples but if you have a feeling that they are increasing more over time then it is not a waste of time to seek the advice of a professional who will tell you to either ignore them and you'll grow out of them of he'll put you on some kind of medication which will help you control and get rid of them.
    it wouldn't hurt to see a derm but it might not help either. at your age they'll probably write it off as puberty. but they might give you some sort of medicine for it. i would just ask your parents to take you if you think it's that big of an issue. like i said, it can't hurt.
    Even if the dermatologist says everything is normal and gives you advice you already know and are doing, it still is worth to seek professional help just for our own peace of mind.

    Can you please give me some advice on going to a Dermatologist?

    I am 14 and i have average acne for a teen but i really want to go see a dermatologist about it. Do you know how much it is to go see one? A session? Does it help?/Did it?Can you please give me some advice on going to a Dermatologist?
    do you have insurance? They are not a cheap visit. I have found that the Dermo. did not tell me anyting that I alread know about my acne. She told me to use Purpose soap and Purpose lotion. She also told me to use Dove for sensitive skin soap, and Aveno oakmeal soap. You can seach for answers already touching this subject. Unless you have really bad acne, I would save your money and try over the counter products.

    What happens at a dermatologist appointment?

    I'm going to the dermatologist for acne and I want to know if they just look at your skin and ask you questions? Also, if i have medication to clear my face would it work on body acne too and if I had medication for body acne would it clear my face also? Thanks!What happens at a dermatologist appointment?
    They bring you into a room and sit you on a chair (almost like a dentist chair) and examine your face. They ask you what products you use on your face, if you have oily, dry, or combination skin, and they will probably address the t-zone. then they'll most likely prescibe you to some expensive products with extremely high percentages of benzoyl peroxide. it made my face burn, peel, and turn red. sometimes they recommend products that are too harsh. and about the medication, ask the dermatologist!

    How many years do have to study to become a dermatologist?

    Which institution do you recommend should i join to become a dermatologists?How many years do have to study to become a dermatologist?
    Dermatologist are doctors. You need to go to college, then medical school. A safe bet is to plan for about 8 years of college. You should take human biology as a college major. Once you get your bachelor's degree, you can pick from thousands of medical schools around the country.How many years do have to study to become a dermatologist?
    A dermatologist is a physician and must complete medical school, a internship, a residency and specialized dermatology training. With your undergrad work taking four years, your total training would be approximately 8-10 years.
    7 years for becoming GP %26amp; 4 to dermatology
    Dermatologists are medical doctors. To get an M.D., you must go through undergraduate school (4 years), medical school (4 years), and multiple years of residency. Specializations such as dermatology are not picked up until late in this process.
    A minimum of 11 years of college and post graduate training is required to become a dermatologist in the United States and Canada.
    i think its 4 yrs at university
  • moon drops
  • Friday, August 20, 2010

    How much do you think it would cost to see a dermatologist, I need to find out if I have skin cancer.?

    your family doctor can assess you. if this is a question posed due to lack of insurance, contact your local American Cancer Society for resources.How much do you think it would cost to see a dermatologist, I need to find out if I have skin cancer.?
    first you should ask yourself why you think you have skin cancer. Then i would advise you to stop thinking about such negative things. Cancer starts in the mind.How much do you think it would cost to see a dermatologist, I need to find out if I have skin cancer.?
    depends on the amount that your insurance covers.


    but that shouldn't matter


    skin cancer can be life-threatening


    so go see a dermatologist!!!
    if you don't have insurance and have to pay out of pocket. I would estimate at least $75. that's what I payed for a visit when I didn't have insurance back in '97 back east by Philly
    Hi from France 鈾?br>




    You know ? Your health is more important than money, then see a doctor please !! Never mind for the price !! OK ?


    I know, it's very easy to say that for french people because we have a good system for the health ( S茅curit茅 Sociale) but all the same !!





    Have a nice day and take care !





    Cat.
    just go to your regular doctor. they can do the same tests.
    THIS COULD BE CANCER. GO SEE A DERMATOLOGIST
    it depends upon the doctor. just ask someone you trust to help you find a great doctor to examine your skin condition. laboratory exams will cost too.
  • eyeshadow
  • Does anyone know how to get veterinary help for a chihuahua who needs to go to a dermatologist in Ohio?

    I have a little chihuahua I took in several years ago from someone who did not want him. Recently he has developed a skin allergy, causing him to be bald from the neck down, on the inside of his legs, and now patches on his body. My vet cannot figure it out, and we have tried several medications, and taking away possible causes. Now we are told he needs to go to a ';dog dermatologist.'; After all the expenses we have already incurred, we are now looking at around $350 to $400, or more. This is really not an expense we are prepared for, as we already spent a bunch on our 3 cats- 1 who was diagnosed with FIV. I was just wondering if anyone knew of any organizations who will help- I have not found any we qualify for. I feel bad for my little guy, but I really need help with the cost. Any ideas would be great- and please be serious. Thanks.Does anyone know how to get veterinary help for a chihuahua who needs to go to a dermatologist in Ohio?
    You can apply on line for this special no interest credit card for veterinary care..All of the vet s in my state accept this, and you have up to 18 months to pay. Most vet clinics will have the form as well..and, you will know in just a couple of minutes if you are accepted...all it requires in a steady income.. http://www.carecredit.com/practices/vete鈥?/a>


    if you are not approved for this credit, you might try this recipe that is excellent for such allergies. A canine dermatologist recommended it for a friend's dog, who had a similar chronic problem..


    A diet of lamb, rice and kelp often helps.





    Cook the white rice with 3 cups of water per cup of rice, and cook until all is absorbed. Grind or chop the lamb and feed it raw, mixed in the rice with sea kelp... give this a try for 3 weeks. If there is even any improvement, the begin adding vegetables.no tomato, corn or onion, but other veggies are ok..peel all potatoes..





    my friend's German shepherd went through the same thing....seemed to have no hope, but that diet finally gave him relief.





    Most store bought meats have growth hormones, and antibiotics, given during the growing or 'finishing' feeding. but lamb is not fed that stuff.





    The kelp makes a big difference.





    If your dog doesn't like the rice, you can boil fresh fish (not farmed) and pour the broth over the rice.





    By the way, vets often over-look the household possibilities..often the allergen is carpet cleaners, or upholstery shampoo..or air fresheners..Swiffer cleaner has been a common problem since it came out recently..Does anyone know how to get veterinary help for a chihuahua who needs to go to a dermatologist in Ohio?
    all u need to do is put a pickle jar that sayed help. And explain what happend
    Wow that is a tough question! Sorry to hear about your pup. You can contact your local ASPCA (animal shelter)and explain your situation. Maybe they will help cover some of the costs. I know for sure that most of them have a spay / neuter program that they run for free or at least minimum cost, maybe they can point you in the right direction....... Hope this is of some help.
    I had one same way took him to couple different vets they said he was allergic to fleas not much I could do but Hartz hydrocortizone shampoo helped alot!
    I had a similar problem with a chi cross. All it took was to switch him to homemade dog food. He was allergic to the ingredients in most types of dog food, and it was easier to simply take him off all of them rather then figure out which ones are safe. Do a search for home made dog food recipes and freeze the meals in zip lock baggies
    All I can say is go out here http://www.shirleys-wellness-cafe.com/





    She has all natural remedies to help people and animal alike.--Best of luck.

    I have really bad acne should I go to a dermatologist?

    I have tried several products including Proactiv and all it did was make me break out more. Is there a safe way to pop zits?I have really bad acne should I go to a dermatologist?
    I would defionitly go to the dermotologist.


    At first I really didnt want to go because I didnt think it would do anything but they can see how sensitive your skin is and the best way to take care of it. my skin is the best it has ever loojks due to my dermatologist.


    hope I helped


    :]I have really bad acne should I go to a dermatologist?
    Yes, you should go see a dermatologist. Popping the zits will not keep new ones from coming in. Visit this guys' website in the meantime. I think what he says makes a lot of sense, and it worked (somewhat) for me. Good luck.





    http://acne.org/
    yes, go to a dermotologist. the safest way to pop a zit is carefully with a metal popping instrument(?)=) ive done it before, its a small metal stick wit kind of a spoon like swoop on the end that you gently pop it with to make sure its clean.


    hope that helps=)
    be careful popping zits because your skin could scar! go get a facial they can do it safely for you. as far as going to a dermatologist, they will offer you a few things anti-biotics that will make you totally sun sensitive and you may have to be on them for a long time. or they can give you accutain which is a really harsh oral medicine that causes your whole body to peel.


    I would go to a clinical skin care therapist first, see if they can treat you, their methods will be more natural and you may not have to commit to a harsh regime.


    as far as Proactive is concerned it's really bad stuff, it basiclly stripped your skin so much that you were making tons of oil, just to lubricate your skin and your skin was holding on to all the water and oil so your cells wern't dieing and falling off the way they are supposed to. that is why your skin got worse.


    good luck
    Do not pop zits! That's what my dermatologist told me. I've been going to the dermatologist for a little over a year now and its been the best decision ever. I don't know why I put it off. I think I thought that I wouldn't be able to afford the medication. I was wrong though - your doctor can prescribe generic medications. You definitely should go, you won't worry about your skin so much once you get the right help.
    No, popping zits is not any better than just letting them be.


    You should really go to a dermatologist. They will probably only tell you what you know, that you have sensitive skin and people get acne in their teen years. But, after you listen to all that boring information, they give you medication for you acne and it really helps. Better than anything that you can buy over the counter.





    Good Luck!
    you should go see a dermatologist ASAP

    What is the best place to see a really good Dermatologist?

    in NYC? Preffably the Bronx. ThanksWhat is the best place to see a really good Dermatologist?
    look for a dermatologist who is board certified under your listings. sorry if you were in chicago i would have the place for you.

    How to talk my parents into taking me to the dermatologist?

    for really bad sweating- doctor reccomended


    and for bad acne-its getting pretty out of control and proactiv sucks.





    what to do?How to talk my parents into taking me to the dermatologist?
    Tell them you keep feeling uglier than everyone else because you have acne all over your face and you realy want to go to the dematologist. Also tell them that your acne hurts when you touch it and its getting anoying.





    Once you are in the dermotolgists office tell the doctor about your sweating problems too.How to talk my parents into taking me to the dermatologist?
    Tell your parents that you are concerned about the increased sweating. It could be that your acne is coming from the excessive sweatiness leaving oil on your skin. Tell them that you are concerned that with the amount of acne you have that you will have permanent scarring and that you want to prevent that from happening by going to the derm.
    PRINT THIS AND GIVE IT TO THEM:





    Hyperhidrosis or excessive sweating is a common disorder affecting many people. Palmar hyperhidrosis or sweaty palms is the most common form of hyperhidrosis, causing excessive sweating of the hands.





    Hyperhidrosis can also cause excessive foot, underarm and facial sweating. It is thought that hyperhidrosis is a result of over activity of the sympathetic nervous system.





    Sweating is often uncontrollable, embarrassing and not anticipated . Normal sweating is needed for thermal regulation however in people suffering from hyperhidrosis, sweating exceeds the bodys need for physiological thermal regulation.





    Hyperhidrosis is an inherited problem that has been seen in almost every ethnic group. People don鈥檛 get used to living with hyperhidrosis but they continue to suffer throughout their lives from it.





    Hyperhidrosis can have severe physiological consequences such as cold and clammy hands, dehydration, and skin infections secondary to maceration of the skin. Hyperhidrosis can also have devastating emotional effects on one鈥檚 individual life.





    Affected people are constantly aware of their condition and try to modify their lifestyle to accommodate this problem. This can be disabling in professional, academic and social life, causing daily embarrassments. Many routine tasks become impossible chores, which can psychologically drain these individuals on a constant basis.





    Hyperhidrosis is a medical condition with severe negative psychological consequences. Unfortunately friends, relatives of affected individuals and even most doctors usually do not understand the tremendous negative impact that this condition can have on one's life.





    Fortunately with the recent advances in medicine, many forms of treatment are available to treat hyperhidrosis. Medications, topical antiperspirants, iontophoresis, botox and surgery have been tried to treat hyperhidrosis. Endoscopic thoracic sympathectomy or ETS is the most effective method to treat hyperhidrosis. The California Institute for Hyperhidrosis and Facial Blushing provides a comprehensive site about hyperhidrosis and the ETS procedure.












    If you have decent health insurance it will only be $15 for the office visit. Often times the dermatologist will give you free sample medicines/creams and write you a prescription. It will be about $10-15 for each prescription.
    just ask your Mom and Dad to remember when they were young and growing up and didn't want there face or body to break out,you may need an antibiotic and a cream to help give you a jump start of getting rid of it.
    it's genetic. it could be due to many factors, such inlude too much fat under skin, skin pour are big, or u eat too much salt. these are may be some problems related to sweating. but if u sweat while just standing then see a doctor.





    luk
    Well in my family it was geneitic and I just came out and said mom or dad to a dermotolgist My proactiv isnt working and she will understand if she or he got acne. If they didn't get acne ask them anyways its not a big deal trust me
    just tell them that you are concerned about your health and that they should be too.


    and then bring up your ';amazing healthcare plan'; and remind them that it won't cost much (or even any money) out of pocket...


    hope this helps.
    tell them if you really want to take this big step by actually going to a doctor, you really need/want it, and these problems are seriously bothering you.
    Call and make your own appointment and tell them when it is. Have your list of facts if they question you about it.

    Which carrier pays more Dermatologist or Orthodontist?

    How many hours do each one work, and what is their salary? Is it possible to make a million? How many years in education does it require? Please no stupid answers.Which carrier pays more Dermatologist or Orthodontist?
    I don't know the careers too deeply, but I think orthodontics is more of a promising career (assuming that you like both). Orthodontists are high in demand, since graduate schools keep limited graduates. Also, orthodontists can run their own clinics which result in a higher salary. I think both professions require the same amount of schooling, however orthodontists probably don't have to work long shifts. I don't really think it's typical to earn more than a million each year for either job.

    How much does professional blackhead extraction usually cost? by a dermatologist?

    I have a TON of blackheads and clogged pores all over my nose, and the sides right beside my nose, and all my pores are big too. I've tried every kind if over the counter blackhead remover I've even gone to the dermotologist for medicine for my acne(but not blackheads)How much does professional blackhead extraction usually cost? by a dermatologist?
    for acne,fair complexion,tanning,sunburn,marks,


    spots,make up tips,hair issues n other skin n hair problems


    u can refer to dis website


    http://skincaretipps168.notlong.com


    it has got so many tips n free beauty samples


    for curing acne and all your skin problems
  • moon drops
  • Is balding reversible. My dermatologist has prescribed me Finax 5mg. Can a higher dosage neutralize juicing?

    If 5mg of finax is enogh for normal circumstances, can a higher dosage i.e 15 mg or soprevent hair from falling even if one is taking Anabolic steroids?Is balding reversible. My dermatologist has prescribed me Finax 5mg. Can a higher dosage neutralize juicing?
    sorry balding is not reversible unless you lost your hair due to a trauma like chemotherapy





    and yes finax is an anti androgen so will neutralise the effects of the juice





    get off the juice mate - save the hair you have remaining and save yourself from eventual impotence, acne and rasin sized b@lls





    with really smart training, rest and nutrition you can get good gains and not have the risk of using gear








    I am a drug free trainer - I can bench 490 and squat 810 lbs and weight 285 lb at 9 % bodyfat

    Going to see a dermatologist to prepare for wedding?

    I'm getting married in November and Im just wondering when I should start seeing a derm to get on a good skincare regimen and start getting facials and that kind of stuff? I have occasional breakouts.. mainly in the chin area.. and I have large pores next to and on my nose.. and my skin seems to get irritated easily.. even with moisturizers like cetaphil and oil of olay for sensitive skin. Any thoughts, stories, or opinions would be greatly appreciated!!Going to see a dermatologist to prepare for wedding?
    I don't think it's necessary to see a dermatologist for a few breakouts, but it's your money to spend. If you do go, start way in advance in case they really mess up your skin. You want to give it time to heal.





    As for skin irritation, stop using all the lotions on your face. You really don't need them either. For large pores, every night wipe your nose and next to it with a few cotton balls with rubbing alcohol on them. Just don't get in your eyes or nostrils.Going to see a dermatologist to prepare for wedding?
    worry about the big stuff not the small ones
    It sounds like a good idea, just don't overspend! Lol!
    Most treatments for acne take a minimum of 3 months to really see consistent results. Also remember that some of the treatments such as the oral antibiotics make you more prone to sunburn, so if you're planning a tropical honeymoon that may be a problem. And some acne meds require that you not get pregnant for many months even after discontinuing them. But a good dermatologist can take all these factors into consideration and set you up with the best skin care regimen and treatments.
    3 to four months b4... gives you time to try different things
    start going as soon as possible... that way you can make your skin healthy and get it reagular and what not just make sure not to do any peels or anything at least 2 weeks before your wedding. the dermatologist will tell you what creams and stuff you need for your skin just ask a lot of questions.
    the sooner you start the sooner you'll see results. also, dermatology isn't an exact science, and your regimen will need to be tweeked to get it to do what it needs to do. you may have some minor set backs at first, so doing it sooner rather than later would be a good idea. good luck!
    I LOVE proactiv, I only had the occasional breakout and it has made my skin flawless! I did take about 2 months for it to ';fully'; work
    Honestly, I would try ProActiv - just give it a month even. I went to the dermatologist for years (probably 10 years) and there was obviously improvement but I always complained to the doctor that I wanted it to be ';perfect'; if I was using so many products... when I started using ProActiv instead of my other prescriptions it was the huge difference I was looking for. I will admit I don't know anything about facials and their results, but if you are going to go to the dermatologist I'd start as soon as possible because a lot of times you'll have to make alterations to the program and he'll want you to try something for a month or 6 weeks before deciding if it's what you need. Hope I helped :-)
    The only thing I know is that I have been told that you should not get a facial right before the wedding...not sure why...thats all I know!!!

    Can I see a dermatologist with Medicare?

    I have a family doctor, and I was going to ask her if I can see a dermatologist with Medicare or if she could recommend one to me.Can I see a dermatologist with Medicare?
    You can, but I think it only covers certain ones. Of course, your doctor can give you a list of those.Can I see a dermatologist with Medicare?
    why u er not try the place as follows. it accomodates more medicare information.

    How do you cure acne w/o prescriptions, antibiotics, dermatologist?

    try some hair skin and nail vitamins - I use that clearasil acne clearing scrub - just so I don't get it. It's got the exfoliants that scrub all the dead skin cells off your face. Keep your face washed and also benzoil peroxide helps.How do you cure acne w/o prescriptions, antibiotics, dermatologist?
    You don't cure it, you treat it.


    My daughter had bad acne and tried a lot of things, including dermatologists.


    What helped her the most ultimately, was cleaning her face everyday with olive oil or coconut oil. This may sound odd, since acne is associated with oily skin, but it cleared her skin wonderfully. I wish I had known about it!How do you cure acne w/o prescriptions, antibiotics, dermatologist?
    i don't know. a healthy diet and drink 10 glasses of water a day.

    How many years of college will I have to attend after highschool to become a dermatologist?

    A dermatologist is an MD, so you must attend 4 years of college than 4 years of medical school. Then you must do a year long internship and then apply for a 3 year residency in dermatology which, along with radiology, opthalmology and anaesthesiology, is incredibly difficult to get.





    In undergrad, you must have a high GPA and MCAT to gain admission to medical school. In medical school you must have high USMLE scores, and must be near the top of your class at a good medical school to get the dermatology residency of your choice.How many years of college will I have to attend after highschool to become a dermatologist?
    About 14-15 years. You will have to go at LEAST 4 years of undergraduate school to obtain a Bachelor's degree. Then you will have to attend 4 years of medical school. You will then have to specialize in dermatology and then do an internship and residency in that area.How many years of college will I have to attend after highschool to become a dermatologist?
    Pretty much the same as most doctors since a dermatologist is a doctor. 4 years college, 4 years medical school, 3-4 years resident. I don't know when the dermatology aspect comes in but likely as part of the residency.
    I'm going into Pharmacy, and I have to be in college for 6 years, and I'm not even going to medical school. So atleast 6...a family doctor goes to school for 10 years, and a veterinarian (probably spelt that wrong) goes for even more, because of all the different animals they have to learn about!





    But the pay is TOTALLY worth the time! Good luck!
    Dermatologists are physicians (medical doctors) specializing in the diagnosis and treatment of diseases and tumors of the skin and its appendages. There are medical and surgical sides to the specialty. Dermatologic surgeons practice skin cancer surgery (including Mohs' micrographic surgery), laser surgery, photodynamic therapy (PDT) and cosmetic procedures using botulinum toxin ('Botox'), soft tissue fillers, sclerotherapy and liposuction. Dermatopathologists interpret tissue under the microscope (histopathology). Pediatric dermatologists specialize in the diagnoses and treatment of skin disease in children. Immunodermatologists specialize in the diagnosis and management of skin diseases driven by an altered immune system including blistering (bullous) diseases like pemphigus. In addition, there are a wide range of congenital syndromes managed by dermatologists.





    A minimum of 11 years of college and post graduate training is required to become a dermatologist in the United States and Canada. This includes graduation from a 4-year college in the college they will take Pre-Medicine, a 4-year medical school followed by a year of post graduate training in medicine, surgery or pediatrics (called an internship) after which a physician may apply for admission to graduate dermatology residency training. Dermatology residencies are among the most competitive in terms of admission. Following the successful completion of formal residency training in dermatology (3 years) the physician is qualified to take certifying board examinations (written) by the American Board of Dermatology or the American Osteopathic College of Dermatology. Once board certified, dermatologists become Diplomates of the American Board of Dermatology or the American Osteopathic College of Dermatology AOCD. They are then eligible to apply for fellowship status in the American Academy of Dermatology. Some dermatologists undertake advanced subspecialty training in programs known as fellowships after completion of their residency training. These fellowships are either one or two years in duration. Fellowships in dermatology include pediatric dermatology, surgical dermatology including Mohs micrographic surgery, dermatopathology (pathology of skin diseases) and dermatological immunology.

    What Can A Dermatologist Actually Do For You?

    Like If you go there what do they do?








    also what is better clindamycin {sp} Or acutance? [sp]





    [(for my sister)[She doesn't have That bad of acne just on my forehead and that's it]What Can A Dermatologist Actually Do For You?
    Accutane will not work for your sister because it is for EXTREME cases of acne, meaning it also has extreme side effects. Your dermatologist will most likely suggest an antibiotic and/or the pill.


    They just talk to you and find out your concerns, give you creams/pills to help you out.


    My favorite skin care regimen supplement is MaMa lotion, it made my skin clear and even, which is difficult because I hyperpigment easily. You can get samples at skincarerx.com. If your sister can figure out the best way for her to cleanse, tone, and moisturize, she will have no problem.What Can A Dermatologist Actually Do For You?
    dermatologists are obviously doctors who specialize in the conditions of the skin, treatment for skin-related problems and can advise you how to take care of your skin to keep it healthy. When you first go into the examination room, you will be checked all over for any unusual ';spots'; and problematic areas...body or facial.....and scalp, also. If your facial skin is dry, combination, oily/acne prone, the doctor will advise you in the right care and treatment of it. When I was young, they helped me (acne---antibiotics)and as I got older, they certainly helped me. My doctor saw a suspicious 'spot' on my neck and sure enough it was basal cell cancer. A treatable and curable cancer...if it was not observed though, it would have progressed to something I do not want to think about!


    Antibiotics and accutane are prescribed accordingly to the severity of the acne condition and how your body will respond to it.....
    Well my cousin had really bad acne and went to a dermotologist. He's not acne free. He's face looks amazing. Also I have mind acne and Proactive really got expensive for me. Now I use AcneFree which is a kit just like Proactive and way more affordable. It only cost 20.00 and you can get it any CVS store. You'll see results with like 3 days.
    they can treat acne, reduce the appearance of wrinkles, treat skin cancer, remove moles. It's not just about the face any skin problem, rashes, ezema. Anything to do with skin.
    I tried using ProActive becuase I had some minor acne on my cheeks and forehead but it didn't help. It actually made it worse!


    A couple months after ProActive failed I went to a dermatologist. He prescribed a low dose antibiotic and 2 topical creams. It cleared up my acne like a dream!!! It worked perfectly!!! It was rather expensive between the visits and the perscriptions...but worth it in my opinion.


    I use those topical creams along with cedaphil face wash and moisturizer. Hope this helps!
    ummm when i went to the dermatologist for acne and basically they ask you questions about the problem you have and they prescribe a medication to you. I would say you benzaclin for acne its not for severe acne its for mild acne and it takes about 2 weeks and really clears it up but you have to use it for atleast 4-6 months and its kinda pricey the everyday price is $158.39.


    Hope this helps.
  • moon drops
  • How would this Dermatologist visit be like?

    What if I visit the dermatologist about my problem with dark underarms having my underarms appear darker than the rest of my skin what cream, prescription or procedure would the dermatologist recommend to me? Please if you have had an experience please share thank you :)How would this Dermatologist visit be like?
    possible laser-treatment...thats is when you are older than 13 or otherwise some whitening-cream but ofcourse it depends on cause of your darkening-skin problem,he/she might tell you to take supplements...just go and put your worries at a rest

    Looking for a dermatologist in New Jersey?

    I am looking for a dermatologist that is familiar the use of the drug Spironolactone for acne treatment and is located in the Northern New Jersey area.Looking for a dermatologist in New Jersey?
    lilly, I acne problems beginning when I was in high school. I had acne on my back that was really embarrasing. I tried Proactiv, Murad, Aceneticin, and even the Asso Gold Cleaning Bar with dismal results. I wound up visitng a local dermatologist in Iowa. After 3 months nearly all of my acne cleared up. I recommed you try finding a New Jersey dermatologist that can help. http://www.homeacnecare.com/New-Jersey-D鈥?/a>Looking for a dermatologist in New Jersey?
    Look it up on yellow pages.

    Do i need a referal from my Doctor inorder for me to see the Dermatologist?

    i have a United Health insurance and i would like to see Dermatologist for my acne, but do i first need to have a referal from my familey doctor or can i just set up the apointment with any Dermatologist office? thanks in advance for your replies.Do i need a referal from my Doctor inorder for me to see the Dermatologist?
    HMO plans require a referral. PPO (Preferred Provider Option) usually does not. It would be best to check your policy.


    -MMDo i need a referal from my Doctor inorder for me to see the Dermatologist?
    Call your insurance company and ask if you need a refferal.
    Yes, but it might depend on what type of insurance you have. I think UHC has two different levels of insurance. I had them for awhile, and almost had to sue them, so make sure you keep track of all your bills and receipts cause there billing department are morons.


    A dermatologist didnt help me when I went. I found this lady (who just happened to be local for me) and she is the only only that helped me after years.


    http://www.facerealityskincare.com/
    you should get a referral as the specialists are busy
    no, you can go directly to a local dermatologist





    if you want to go to a professional derm. then seek referrals to be added to a waiting list

    How much would it cost a for a dermatologist to remove a wart on my knee?

    It's a rather big sized wart almost as big as a dime!!How much would it cost a for a dermatologist to remove a wart on my knee?
    It shouldn't cost more than $80.00 to $100.00 dollars





    Perhaps it might be cheaper to go to your regular physician. Call first and see if he can remove the wart.





    Good luckHow much would it cost a for a dermatologist to remove a wart on my knee?
    You do not need to see a dermatologist.If you go to a chemist there are ointments that you can apply on the wart and after several time doing this it will go.....the ointment will kill it and it will peel off.Just go to a chemist and ask for wart treatments.
    I had a wart on my finger and my Dr. tried burning it off. It kept coming back. Eventually he asked me if I wanted to try an ';unapproved method'; of removing warts. He injected the wart with a flu shot and the wart went away and never came back again.

    What is a cheaper alternative for visiting a dermatologist?

    Depends on whats wrong and how bad it is. if what ever it is looks serious go to a dermatologist.What is a cheaper alternative for visiting a dermatologist?
    put alcohol on ur face. bud light works best :]What is a cheaper alternative for visiting a dermatologist?
    Depends what's wrong but for the most part you can do your own research and learn much about your condition. We went to 8 different doctors to no avail then started doing research and found many natural solutions to the problem.





    If you have something serious definitely get it checked out so you have a starting point.
    not going

    How much does it cost to see a dermatologist?

    I have severe acne and it WILL NOT go away, iv'e tryed pro activ and anything else you can name, its JUST started to decrease a little now, but i'd like to see a dermatologist about it. I live in brisbane , qld, AUS. And are they certain to get rid of my acneHow much does it cost to see a dermatologist?
    Hopefully you have health insurance.....How much does it cost to see a dermatologist?
    dont worry i have really bad acne, i went to my doctor and he gave me some lotion that seems to be working. urm its about $1200 for the whole face i think x good luck : )
  • moon drops
  • Is my dermatologist right on facial skin care?

    M dermatologist told me that when the skin on our face becomes dry and flaky, we should NOT apply moisteriser as that makes the flakes of dead skin stick back down and clogs the pores. Instead, she said it is better to apply nothing to the face that night, so the skin can renew itself or to apply a very light serum. I was astonished as I thought the answer to dry flaking skin was to moisterise. Is she right or is she talking nonsense? I thought a serum moisterised anyway?Is my dermatologist right on facial skin care?
    Your dermatologist is right. Moisturiser is to stop dry flaky skin from occurring in the first place not to cure it. If your skin is flaking moisturiser won't cure it, you have to wait for the skin to flake off. Your skin renews itself naturally while you sleep. Gentle exfoliating helps, but be careful not to over exfoliate as this can irritate the skin underneath.Is my dermatologist right on facial skin care?
    She might be right, but I never heard of such a thing really. I would use some gentle face scrub, and cream for my face type.





    My skin was very dry, and I fixed the problem with good creams in short time. First I used Murad, which was pretty expensive, then I changed to seba med, which are very good products as well. Check more info on link below.
    Mostly true. ALL skin needs moisturizer, even oily skin, because when you wash your face, you're depleting it of moisture it needs to repair itself.





    The best thing for you to do for dry, flaky skin (as long as your skin isn't terribly sensitive) would be to get a product with a low level of glycolic acid. It will make you peel quite a bit initially, but it will remove the dead skin and aid in the healing process.





    The next thing is to get on a good skin care regimen. Avoid harsh gel cleansers and stick to hydrating creamy cleansers. Aveda makes a great line for dry skin and it's all natural, too.





    Serums can be a bit different from moisturizers, actually, depending on which type you're talking about. Serums are a more concentrated direct treatment, but are generally not designed to be used all over your face as a daily treatment because they can clog your pores.





    There's also another great line by Peter Thomas Roth. They have a product called Botanical Buffing Beads, which is the best exfoliant I've ever used on myself or anyone else. The exfoliating beads are made of jojoba, which break down with water and actually become a second wash to help clean off your skin of the dry skin and impurities the exfoliation just removed.
    dermatologist cant be wrong


    //